Errors in Reasoning Questions - - Question 32
Naz October 11, 2013
The question stem for this question is: "Which one of the following describes a flaw in the reasoning underlying the foundation's conclusion?" The conclusion of the stimulus is: "the weapons research would not benefit from the foundation's grant." Why? Because the university promised that none of the foundation's money would be used for the weapons research. What the foundation is ignoring is the possibility that funds that might have gone towards humanitarian uses can now be funneled towards weapons research, since the money from the foundation can then replace this newly diverted, formerly humanitarian money.Rob Dixon October 11, 2013
Naz, I am interpreting that the University is making the conclusion, not the foundation (as the question-stem suggests), because the thing that the foundation ever does in the passage is threaten to pull funding, not conclude anything. I was hoping you could look at the question stem and determine if the error of reasoning should belongs to the foundation or to the University. I am not disputing as to whether choice A is correct, since it is the only one that makes any sense. Just that I believe there may be a type-o in the question stem.Rob Dixon October 11, 2013
Naz, disregard my last post. The clause in that is interjecting in the middle of the last statement assigns the conclusion to the foundation. I understand where I went wrong. Thanks.